LSAT and Law School Admissions Forum

Get expert LSAT preparation and law school admissions advice from PowerScore Test Preparation.

 Administrator
PowerScore Staff
  • PowerScore Staff
  • Posts: 8917
  • Joined: Feb 02, 2011
|
#40053
Complete Question Explanation
(The complete setup for this game can be found here: lsat/viewtopic.php?t=6800)

The correct answer choice is (B)

If R is the first piece, then the second rule would force the following relationship between R, S, and T:
PT73_Game_#1_#3_diagram 1.png
Similarly, to satisfy the third rule we need to ensure that W is later than both R and T:
PT73_Game_#1_#3_diagram 2.png
These two rules, when combined with the first rule (S V), establish the following sequencing chain, which—you may notice—is consistent with Template 2B:
PT73_Game_#1_#3_diagram 3.png
Given the local requirement that R is the first piece on the CD, we can create the following basic linear diagram to represent the assignment of variables to the base:
PT73_Game_#1_#3_diagram 4.png
The only variable whose position is relatively unknown is V. Because you are solving a Could Be True question, you should expect that the correct answer choice will exploit this uncertainty—a prephrase that immediately shows answer choice (B) to be correct. None of the other four answer choices address the relative position of V, and none of them could be true.

Answer choice (A): This answer choice cannot be true, because the second piece must be S, and T is either 3 or 4.

Answer choice (B): This is the correct answer choice, because the placement of V is uncertain relative to S, T, and W.
Consequently, V could be third:
PT73_Game_#1_#3_diagram 5.png
Answer choice (C): This answer choice cannot be true, because W must be either 4 or 5.

Answer choice (D): This answer choice cannot be true, because S must be 2.

Answer choice (E): This answer choice cannot be true, because W is later than T.
You do not have the required permissions to view the files attached to this post.
 cnoury1221
  • Posts: 15
  • Joined: Jun 24, 2019
|
#65957
Hello,

I was confused about the explanation for answer choice A here: How can the second piece be either S or V? Using template 2b. and implementing the questions condition ( R is the first piece), shows that "S" must be second?

I appreciate clarification here!

Thank you!

Carolyn
 Adam Tyson
PowerScore Staff
  • PowerScore Staff
  • Posts: 5153
  • Joined: Apr 14, 2011
|
#65965
You are absolutely right, Carolyn! We will correct that portion of the explanation. Thanks for pointing that out!

Get the most out of your LSAT Prep Plus subscription.

Analyze and track your performance with our Testing and Analytics Package.